Hamiltoniano del oscilador armónico cuántico con ψ(x)=δ(x)ψ(x)=δ(x)\psi(x)=\delta(x): comparación con la mecánica clásica

Estaba leyendo la pregunta ¿Por qué no puedo? ψ ( X ) = d ( X ) en el caso de un oscilador armónico? La respuesta aceptada dice que ψ ( X ) = d ( X ) es un estado matemáticamente válido, aunque no es físicamente posible porque las distribuciones delta no son normalizables. Si es posible, me gustaría ignorar el hecho de que no es físico y, en cambio, tratar de llenar los vacíos en mi comprensión del modelo matemático.

La respuesta concluye que el valor esperado del hamiltoniano del sistema es infinito, lo cual tiene sentido para mí porque se sigue de la proposición que pag 2 es infinito.

Sin embargo, veo esto como una contradicción de mi impresión de que los sistemas mecánicos cuánticos a altas energías se aproximan fácilmente al comportamiento clásico. Por ejemplo, con una partícula en un pozo cuadrado, la distribución de probabilidad de posición a altas energías es aproximadamente constante en toda la región permitida, que es exactamente lo que predice la física clásica para la misma energía.

Volviendo al caso de los osciladores armónicos, creo que un modelo clásico con energía cero tendría una distribución de posición de d ( X ) , porque la partícula está localizada en un lugar exacto y no oscila ni se mueve de ninguna manera. Esto me dice que un oscilador armónico cuántico con una energía extremadamente alta se comporta como un sistema clásico de energía cero, lo que generalmente suena mal.

¿Es incorrecto/sin sentido hacer tales comparaciones de una partícula clásica de energía cero con un oscilador armónico cuántico con ψ ( X ) = d ( X ) ? Alternativamente, ¿es esta aparente contradicción realmente justificable lógicamente?


Creo que hay algo sospechoso en mi comparación cuando digo que un sistema clásico muestra una distribución de posiciones de d ( X ) , ya que la distribución de cantidad de movimiento también es una función delta (con un pico en pag = 0 ). Sin embargo, el manejo del impulso QM predeciría un estado completamente diferente. Pero no puedo construir más a partir de ese razonamiento para ver si es relevante.

Además, el estado de energía más bajo de un oscilador QM es distinto de cero, por lo que no hay una contraparte QM válida de un oscilador clásico con ψ ( X ) = d ( X ) . Sin embargo, esto también es cierto para la partícula en un pozo infinito, por lo que esperaría una simetría en la que, en ambos casos, las descripciones de QM de alta energía sean similares a la mecánica clásica.

Como referencia, la respuesta mencionada en la descripción de la recompensa es physics.stackexchange.com/a/457479 . Lo siento, olvidé ese detalle.
No es correcto asumir que una partícula con alta energía siempre puede ser descrita por la mecánica clásica. Considere por ejemplo dos estados | ψ 1 > y | ψ 2 > , en el que la partícula tiene una alta energía. Cualquier superposición de ellos es un estado válido en mecánica cuántica, pero no tiene sentido en mecánica clásica.
Por cierto, ψ ( X ) = d ( X ) no satisface la ecuación de Schrödinger, por lo que no puede ser el estado fundamental.
@atarasenko Su primer comentario sería una muy buena adición a su respuesta (o tal vez sea una segunda respuesta). Es un buen contraejemplo para mostrar que la comparación no es válida.
He agregado información del comentario a la respuesta.
Una forma directa de ver eso. d ( X ) es el niño del cartel de la profunda no clasicismo de un estado cuántico es que es literalmente una superposición coherente de todos los estados propios de momento con cada uno teniendo pesos iguales (igual probabilidad-amplitudes)! Como explica @atarasenko, el criterio de clasicismo debería ser minimizar la incertidumbre tanto en la posición como en el momento simultáneamente y esto se logra mediante los llamados estados coherentes (De hecho, la teoría de los estados coherentes es muy sutil y elaborada y debería ser divertido comprobarlo!).
Si se me permite, creo que el impulso de su parte de tratar d ( X ) como un estado de tipo clásico surge de pensar que al contener la partícula para estar en una posición específica en lugar de en una región distribuida de forma "borrosa", d ( X ) logra introducir de contrabando en clasicismo a la historia. [...]
[...] Pero esto no es cierto porque la naturaleza de la mecánica cuántica no radica en restringir que un observable tenga un valor nítido, sino que radica en la no conmutatividad de diferentes observables. Es decir, el observable puede tomar un valor tan preciso como quiera, pero sus socios que no viajan tendrán que estar distribuidos de manera conmensurablemente borrosa.

Respuestas (4)

Los estados más clásicos del oscilador armónico son Estados coherentes del oscilador armónico , no d ( X ) . La razón es que el estado coherente tiene una incertidumbre equilibrada de coordenadas y cantidad de movimiento:

Δ X = 2 metro ω ; Δ pag = metro ω 2 ,
y en el límite clásico ( 0 ) Δ X 0 , Δ pag 0 .

en un estado con ϕ ( X ) = d ( X ) la incertidumbre del impulso Δ pag = , no tiende a cero.

ACTUALIZAR:

No es correcto asumir que una partícula con alta energía siempre puede ser descrita por la mecánica clásica. Considere por ejemplo dos estados | ψ 1 y | ψ 2 , en el que la partícula tiene alta energía. Cualquier superposición de ellos es un estado válido en mecánica cuántica pero no tiene sentido en mecánica clásica.

Tus intentos de tomar un límite clásico realmente no tienen sentido. Por la naturaleza de la mecánica clásica, fuera de la mecánica estadística, no existe una "distribución de posición" de un sistema clásico. Un sistema clásico siempre tiene una posición y un momento definidos, por lo que sus "distribuciones" de posición y momento serían d ( X ) y d ( pag ) , respectivamente. Eso es completamente cierto independientemente del estado específico del sistema clásico o de cualquier otra propiedad del sistema, por lo que no es un enfoque útil para pensar qué sistema cuántico corresponde a este.

2. Existe una distribución de posición en un sistema clásico, al menos para estados ligados. Solo tiene que promediar sobre el período de movimiento. Entonces la distribución de posición cuántica irá a una clásica en el límite apropiado.
@Ruslan No entiendo cómo se supone que "promediar" durante un período de movimiento produce una distribución. ¿Quiere decir que desea muestrear aleatoriamente de manera uniforme la posición de la partícula en el tiempo y que, para el movimiento periódico, la distribución de probabilidad de la posición clásica en ese caso es la misma que la de algún estado cuántico? ¿Qué tipo de estado cuántico? (Ciertamente no puede valer para todos ellos, y no está claro a priori cómo tomar el límite clásico de un estado )
Sí, fui descuidado al formular esto. Quise decir exactamente esto. Y creo que no es el límite de un estado, sino el límite de una secuencia de estados. Por ejemplo, elegir un conjunto de cantidades conservadas para definir sin ambigüedades un estado clásico. Ahora construya su contraparte cuántica "más cercana" (suavizando todos los valores exactos para tener incertidumbres razonables, dependiendo de ). Entonces toma el límite 0 . Sin embargo, admito que comencé desde el estado "objetivo" clásico aquí, y estoy de acuerdo en que no todos los estados cuánticos pueden llevarse de manera significativa / única a un límite clásico.
Encuentro que el primer punto en esta respuesta es extremadamente falso. De hecho, es correcto señalar que d ( X ) no es un elemento de L 2 ( R ) . Sin embargo, es completamente engañoso dar la impresión de que no hay formas de arreglar eso (cuando realmente existen ) y ocultar cualquier mención de esas soluciones. Esto es particularmente notable dado que ACM ha reconocido explícitamente que conocen esas soluciones y solo se descartan por preferencia personal.
@EmilioPisanty Usar un espacio de Hilbert amañado no significa que sea válido aplicar la versión estándar del principio de incertidumbre al | X objetos. De hecho, ni siquiera está justificado aplicarlo a todos los estados en L 2 ( R ) porque X y pag solo se definen en subconjuntos densos y el rhs ( [ X , pag ] ) solo existe en la intersección de los subconjuntos sobre los que se definen. Todo lo que hace el uso de un espacio de Hilbert manipulado es permitirle establecer rigurosamente qué tipo de objeto | X es , no te permite actuar como si fuera un estado.
@ACuriousMind Sí. Le invitamos a no estar de acuerdo con las elecciones pedagógicas tomadas sobre la pregunta a la que se ha vinculado, en cuyo caso el lugar correcto para hacerlo es como una respuesta allí. Aquí, su texto simplemente está haciendo un hombre de paja de la respuesta a la que se ha vinculado, y no estoy particularmente interesado en un debate sobre los méritos hasta que se solucione ese aspecto.
Con respecto a "la posición [de un sistema clásico] y las 'distribuciones' de impulso serían d ( X ) y d ( pag ) ", uno podría mirar bien el cuadrado con energía distinta de cero, donde la partícula rebota, por lo que si lo mide en un momento aleatorio , existe una probabilidad uniforme de que lo encuentre en cualquier lugar, y el momento podría ser positivo o negativo con probabilidades iguales. ¿Es impropio comparar eso con QM? Porque estoy bastante seguro de que lo he visto en alguna parte. Tal vez sea significativo que en el caso clásico la partícula está en una posición definida en un momento dado, mientras que es ¿No está en QM?
@Chair El "en un momento aleatorio" es crucial y es a lo que Ruslan se refería en su comentario. Tenga en cuenta que este no es el "mismo tipo de aleatorio" que las mediciones en la mecánica cuántica.

Su identificación de la mecánica clásica como el límite de alta energía de la mecánica cuántica es errónea. Puedo dar varios ejemplos. Considere el estado del oscilador armónico de alta energía (fock/number)

| ψ = | norte = 1000

El 1000 t h estado propio de energía del oscilador armónico. La amplitud del movimiento está bien definida, sin embargo, la fase está completamente indefinida. El oscilador podría aparecer en cualquier posición y con cualquier momento consistente con X 2 + PAG 2 = 1000 (en las unidades correspondientes).

Considere también el estado del gato.

| α = 1000 + | α = 1000

Donde esta es una superposición de dos estados coherentes de alta energía. En este caso, el sistema se encuentra en una superposición macroscópica de dos estados, lo que es muy poco clásico.

Estos son dos ejemplos de estados cuánticos que son de alta energía pero no clásicos.

Creo que es probable que los estados cuánticos de baja energía genéricamente (estados cercanos al estado fundamental) siempre sean no clásicos. Por lo tanto, la alta energía puede ser una condición necesaria para que un estado parezca clásico, pero ciertamente no es una condición suficiente.

El estado que está considerando es X | ψ = ψ ( X ) = d ( X ) . Es incorrecto decir que este estado tiene mucha energía. No es un estado propio de energía y, como se puede ver por una transformada de Fourier, es una superposición de estados con todas las energías posibles. Es decir, tiene aportes de baja energía y alta energía. ¡Tiene energía a todos los niveles! Esto es parte de por qué es un estado altamente no clásico. Solo agregaré aquí que la razón por la que este estado no es físico es porque este estado requiere energía infinita para prepararse y eso no está disponible. Sin embargo, podría preparar un estado que tenga una ocupación uniforme de todos los niveles de energía hasta la energía mi 0 . En este caso, la función de onda sería una función estrecha que se aproxima a un delta de dirac. Tendría un ancho proporcional a 1 mi 0 , por lo que cuanto mayor sea la energía a la que pueda llegar, más cerca podrá aproximarse a la d función.

Ahora la cuestión de una función delta de dirac clásica. Sí, un sistema clásico en CUALQUIER estado determinista (ignorando los estados térmicos/estocásticos) se mostrará como un delta de dirac en el espacio de fase. Tendrá una posición y un impulso bien definidos. Se puede ver que la energía se puede calcular para ser mi = X 2 + PAG 2 (nuevamente con algunas escalas) y, lo que es más importante, que la energía esté bien definida y sea finita.

La gran diferencia es que mecánicamente cuánticamente es imposible tener una función delta tanto en posición como en momento. Simplemente no es un estado válido dentro del espacio de Hilbert. Una forma de ver esto es porque la posición y el momento son pares de transformadas de Fourier y la función delta de dirac no es su propia transformada de Fourier. Esta es la declaración principal de la versión de función de onda del principio de incertidumbre.

Dicho de otra manera, el oscilador clásico de estado fundamental tiene X = 0 y PAG = 0 . No se está moviendo. No tiene energía. Un oscilador cuántico con ψ ( X ) = d ( X ) tiene X = 0 pero tiene TODOS los momentos posibles. Lo que significa que simultáneamente no se mueve (análogo al oscilador clásico) y se mueve infinitamente rápido en todas las direcciones. Esto es muy diferente del oscilador clásico.

Estoy diciendo mucho aquí y siento que me estoy repitiendo, así que tal vez sería mejor que hicieras preguntas aclaratorias.

"Es decir, tiene aportes de baja energía y alta energía. ¡Tiene energía en todos los niveles!" => por lo que la energía "promedio" sería muy alta (infinita). Entonces, el estado de la función delta puede considerarse como un estado de alta energía. Pero me gusta su ejemplo del estado de Fock con grandes cuantos como contraejemplo.
Sí, el estado de la función delta tiene una energía "promedio" de infinito, no estoy en desacuerdo. Elegí expresarlo de la manera un tanto extraña que hice para conectarme mejor con la esperanza de conexión de los OP. ψ ( X ) = d ( X ) a un estado clásico. Pensado en el espacio de fase la función de onda espacial ψ ( X ) = d ( X ) parece el estado clásico que el OP está imaginando. La distribución de impulso se parece a ϕ ( pag ) = 1 . Entonces tiene un componente en 0.
Con ese fin, el estado cuántico ψ ( X ) = d ( X ) "contiene" un componente que "se parece" al estado clásico que imagina el OP. Sin embargo, también "contiene" componentes de mayor energía. Esto es lo que estaba tratando de enfatizar.

Conjuntos de respuestas de Emilio Pisanty = 1 , que esconde algunas sutilezas respecto al límite clásico. con explícito , la transformada de Fourier contiene mi i pag X / , lo que dificulta la interpretación del significado del límite clásico 0 . Es más fácil ver lo que sucede si considera un paquete de ondas gaussianas con una dispersión de espacio de posición Δ X en cambio. Encontrarás que la dispersión en el espacio de momento Δ pag / Δ X . Así que si tomamos Δ X 0 mientras lo esté agarrando arreglado para obtener un d -función posición función de onda, como lo hace Emilio, entonces encontramos que la energía cinética va como ( Δ pag ) 2 2 / ( Δ X ) 2 y diverge. Físicamente, por supuesto, está arreglado, por lo que esto suele ser lo más lógico. Pero en el límite clásico es más útil pensar en tomar 0 antes de tomar Δ X 0 . En este caso obtenemos una partícula clásica "determinista" con ambos Δ X = Δ pag = 0 , que puede tener energía cero.

Todo eso es algo heurístico, porque realmente deberíamos estar tomando proporciones adimensionales a cero. La forma más rigurosa de hacerlo es considerar las escalas características de longitud y momento inherentes al potencial del oscilador en particular, y pensar en los diferenciales de paquetes de ondas en relación con esas escalas. Descubrirá que el límite clásico realmente corresponde a regímenes en los que tanto la dispersión del espacio de posición como la del espacio de momento son mucho más pequeñas que esas escalas características, de modo que la partícula está casi perfectamente localizada tanto en el espacio de posición como en el de momento. En este caso encontramos que la "energía clásica" resulta ser mucho mayor que la energía de punto cero ω , por lo que este último puede despreciarse. Esa es la forma rigurosa de justificar si tomar Δ X 0 o 0 primero; la elección correcta refleja la dispersión de posición relativa de su función de onda a la escala de longitud relevante establecida por el hamiltoniano.

Hmmmm. Parece que esa respuesta tenía un uso inconsistente de ; ha sido editado para completar lo que falta s. No creo que afecte la pregunta o el núcleo de esta respuesta, pero es posible que desee reformular un poco.